Difference between revisions of "2000 AMC 12 Problems/Problem 11"

(Video Solution)
(Solution 4 (Lame))
 
(14 intermediate revisions by 10 users not shown)
Line 4: Line 4:
 
Two non-zero [[real number]]s, <math>a</math> and <math>b,</math> satisfy <math>ab = a - b</math>. Which of the following is a possible value of <math>\frac {a}{b} + \frac {b}{a} - ab</math>?
 
Two non-zero [[real number]]s, <math>a</math> and <math>b,</math> satisfy <math>ab = a - b</math>. Which of the following is a possible value of <math>\frac {a}{b} + \frac {b}{a} - ab</math>?
  
<math>\text{(A)} \ - 2 \qquad \text{(B)} \ \frac { - 1}{2} \qquad \text{(C)} \ \frac {1}{3} \qquad \text{(D)} \ \frac {1}{2} \qquad \text{(E)} \ 2</math>
+
<math>\textbf{(A)} \ - 2 \qquad \textbf{(B)} \ \frac { -1 }{2} \qquad \textbf{(C)} \ \frac {1}{3} \qquad \textbf{(D)} \ \frac {1}{2} \qquad \textbf{(E)} \ 2</math>
  
 
==Solution 1==
 
==Solution 1==
Line 13: Line 13:
 
==Solution 2==
 
==Solution 2==
 
This simplifies to <math>ab+b-a=0 \Rightarrow (a+1)(b-1) = -1</math>. The two integer solutions to this are <math>(-2,2)</math> and <math>(0,0)</math>. The problem states than <math>a</math> and <math>b</math> are non-zero, so we consider the case of <math>(-2,2)</math>. So, we end up with <math>\frac{-2}{2} + \frac{2}{-2} - 2 \cdot -2 = 4 - 1 - 1 = 2 \Rightarrow \boxed{\text{E}}</math>
 
This simplifies to <math>ab+b-a=0 \Rightarrow (a+1)(b-1) = -1</math>. The two integer solutions to this are <math>(-2,2)</math> and <math>(0,0)</math>. The problem states than <math>a</math> and <math>b</math> are non-zero, so we consider the case of <math>(-2,2)</math>. So, we end up with <math>\frac{-2}{2} + \frac{2}{-2} - 2 \cdot -2 = 4 - 1 - 1 = 2 \Rightarrow \boxed{\text{E}}</math>
 +
 +
==Solution 3==
 +
Just realize that two such numbers are <math>a = 1</math> and <math>b = \frac{1}{2}</math>. With this, you can solve and get <math>2 \Rightarrow\boxed{\text{E}}</math>
 +
 +
==Solution 4 ==
 +
Set <math>a</math> to some nonzero number. In this case, I'll set it to <math>4</math>.
 +
 +
Then solve for <math>b</math>. In this case, <math>b=0.8</math>.
 +
 +
Now just simply evaluate. In this case it's 2. So since 2 is a possible value of the original expression, select <math>\boxed{E}</math>.
 +
 +
~hastapasta
 +
 +
== Video Solution ==
 +
https://www.youtube.com/watch?v=7-RloNHTnXM
  
 
== Video Solution ==
 
== Video Solution ==
Line 22: Line 37:
  
 
https://www.youtube.com/watch?v=8nxvuv5oZ7A&t=3s
 
https://www.youtube.com/watch?v=8nxvuv5oZ7A&t=3s
 
https://youtu.be/ZWqHxc0i7ro?t=9
 
  
 
==See also==
 
==See also==

Latest revision as of 09:34, 14 October 2023

The following problem is from both the 2000 AMC 12 #11 and 2000 AMC 10 #15, so both problems redirect to this page.

Problem

Two non-zero real numbers, $a$ and $b,$ satisfy $ab = a - b$. Which of the following is a possible value of $\frac {a}{b} + \frac {b}{a} - ab$?

$\textbf{(A)} \ - 2 \qquad \textbf{(B)} \ \frac { -1 }{2} \qquad \textbf{(C)} \ \frac {1}{3} \qquad \textbf{(D)} \ \frac {1}{2} \qquad \textbf{(E)} \ 2$

Solution 1

$\frac {a}{b} + \frac {b}{a} - ab = \frac{a^2 + b^2}{ab} - (a - b) = \frac{a^2 + b^2}{a-b} - \frac{(a-b)^2}{(a-b)} = \frac{2ab}{a-b} = \frac{2(a-b)}{a-b} =2 \Rightarrow \boxed{\text{E}}$.

Another way is to solve the equation for $b,$ giving $b = \frac{a}{a+1};$ then substituting this into the expression and simplifying gives the answer of $2.$

Solution 2

This simplifies to $ab+b-a=0 \Rightarrow (a+1)(b-1) = -1$. The two integer solutions to this are $(-2,2)$ and $(0,0)$. The problem states than $a$ and $b$ are non-zero, so we consider the case of $(-2,2)$. So, we end up with $\frac{-2}{2} + \frac{2}{-2} - 2 \cdot -2 = 4 - 1 - 1 = 2 \Rightarrow \boxed{\text{E}}$

Solution 3

Just realize that two such numbers are $a = 1$ and $b = \frac{1}{2}$. With this, you can solve and get $2 \Rightarrow\boxed{\text{E}}$

Solution 4

Set $a$ to some nonzero number. In this case, I'll set it to $4$.

Then solve for $b$. In this case, $b=0.8$.

Now just simply evaluate. In this case it's 2. So since 2 is a possible value of the original expression, select $\boxed{E}$.

~hastapasta

Video Solution

https://www.youtube.com/watch?v=7-RloNHTnXM

Video Solution

https://youtu.be/ZWqHxc0i7ro?t=6

~ pi_is_3.14

Video Solution

https://www.youtube.com/watch?v=8nxvuv5oZ7A&t=3s

See also

2000 AMC 12 (ProblemsAnswer KeyResources)
Preceded by
Problem 10
Followed by
Problem 12
1 2 3 4 5 6 7 8 9 10 11 12 13 14 15 16 17 18 19 20 21 22 23 24 25
All AMC 12 Problems and Solutions
2000 AMC 10 (ProblemsAnswer KeyResources)
Preceded by
Problem 14
Followed by
Problem 16
1 2 3 4 5 6 7 8 9 10 11 12 13 14 15 16 17 18 19 20 21 22 23 24 25
All AMC 10 Problems and Solutions

The problems on this page are copyrighted by the Mathematical Association of America's American Mathematics Competitions. AMC logo.png